Q8

 
cvfh17
Thanks Received: 0
Forum Guests
 
Posts: 22
Joined: March 30th, 2013
 
 
 

Q8

by cvfh17 Wed Apr 03, 2013 6:57 pm

can somebody explain this question?? thanks
User avatar
 
tommywallach
Thanks Received: 468
Atticus Finch
Atticus Finch
 
Posts: 1041
Joined: August 11th, 2009
 
This post thanked 1 time.
 
 

Re: Q8

by tommywallach Sun Apr 07, 2013 1:14 pm

Hey There,

Image

So here's a full diagram of this game. We would call this an open grouping game, so we start by making three slots for each illness (because each illness could have all three). We put a box around any slot that must be filled, and because each illness needs "at least one" symptom, we put boxes all along the bottom row. From there, we go through the constraints:

1) Fill in H and S for illness J.
2) This means that K must have symptom F, and also that J can't have all three symptoms. We can thus X out the remaining slots for both J and K.
3) Pause on this constraint for now.
4) This means we need a second box on L, because K only has 1 symptom for sure.
5) This means that L can only have 2 symptoms, and N must have 1 symptom (because if they both had 2, there would be an overlap).
6) This means M must have all 3 symptoms.

Keep the remaining rules to the side, and you're set.

If this is done correctly, this question is crazy simple.

(A) J and N can't be the same, because J has 2 symptoms and N only has 1 symptom.

(B) K and L can't be the same, because L has 2 symptoms and K has only 1 symptom.

(C) K and N both have one symptom, and it could be the same one.

(D) L and M can't be the same, because L has 2 symptoms and M has all 3.

(E) M and N can't be the same because M has all 3 symptoms and N only has one.

Hope that helps!

-t
Tommy Wallach
Manhattan LSAT Instructor
twallach@manhattanprep.com
Image